Fourier Transform - Solving for Impulse Response

Click For Summary
The discussion focuses on solving for the impulse response h(t) using the excitation signal x(t) and output signal y(t) through convolution. The user expresses uncertainty about taking the Fourier Transform of the output signal elements and whether their approach is correct. They correctly apply the convolution theorem and the duality principle, aiming to derive H(f) from the relationship Y(f) = H(f)X(f). The user has attempted to distribute the unit step functions in y(t) and compute their Fourier transforms, seeking validation of their methods. Overall, the thread emphasizes the importance of accurately applying Fourier Transform techniques in signal processing.
Abide
Messages
16
Reaction score
0

Homework Statement


I'm trying to Solve for an impulse response h(t) Given the excitation signal x(t) and the output signal y(t)

x(t) = 4rect(t/2)
y(t) = 10[(1-e-(t+1))u(t+1) - (1-e-(t-1))u(t-1)]
h(t) = ?

y(t) = h(t)*x(t) --> '*' meaning convolution!

I am unsure how to take the Fourier Transform of the elements in the output signal. I have posted my attempts below and I would like to know if I am going this correctly or not, Thanks!

Homework Equations


Using the multiplication - convolution duality I know that we need to take the Fourier transform of each element giving us the following...

Y(f) = H(f)X(f)

Which then allows us to solve for H(f) by Y(f)/X(f)

The Attempt at a Solution



First I distributed the unit step functions in y(t) giving...

y(t) = 10[u(t+1)-e-(t+1)u(t+1) - u(t-1) + e-(t-1)u(t-1)

Now I take the Fourier transform of each element in y(t)

F(u(t+1)) = 1/(jω+(02))(ej2∏f)

F(e-(t+1)u(t+1)) = 1/(jω+(12))(e-j2∏f)

I got this by using the following definition of the Fourier Transform
e-Atu(t) <---> 1/(jω+A2) for A > 0I was curious as to if anyone could give me some insight on whether I am performing these operations correctly or not. I apologize if I left out any information!
 
Last edited:
Physics news on Phys.org
Find the Fourier transforms of x(t) and y(t),then use the convolution theorem and inverse transform.
 
Ok, but my question was more about whether I am performing the transform correctly , thank you for your response
 
Question: A clock's minute hand has length 4 and its hour hand has length 3. What is the distance between the tips at the moment when it is increasing most rapidly?(Putnam Exam Question) Answer: Making assumption that both the hands moves at constant angular velocities, the answer is ## \sqrt{7} .## But don't you think this assumption is somewhat doubtful and wrong?

Similar threads

Replies
1
Views
2K
  • · Replies 1 ·
Replies
1
Views
2K
  • · Replies 6 ·
Replies
6
Views
3K
  • · Replies 3 ·
Replies
3
Views
2K
  • · Replies 7 ·
Replies
7
Views
2K
  • · Replies 2 ·
Replies
2
Views
1K
  • · Replies 3 ·
Replies
3
Views
2K
Replies
2
Views
2K
  • · Replies 1 ·
Replies
1
Views
2K
  • · Replies 9 ·
Replies
9
Views
2K